Mental Health Nursing

Ace your homework & exams now with Quizwiz!

Which client statement alerts the nurse that the client may be maladaptively responding to stress?

"Avoiding contact with others helps me cope."

A client is diagnosed with an anxiety disorder. The nurse counselor recommends the behavioral technique of reciprocal inhibition. The client asks, "What's that?" Which is the best nursing reply?

"By introducing an adaptive behavior that is mutually exclusive to your maladaptive behavior, we will expect subsequent behavior to improve."

A nursing instructor is teaching about the etiology of hypochondriasis from a psychoanalytical perspective. What student statement about clients diagnosed with this disorder indicates that learning has occurred?

"They express personal worthlessness through physical symptoms because physical problems are more acceptable than psychological problems."

The nurse is providing information on self-help groups like Alcoholics Anonymous (AA) to a client. Which client statement indicates that learning has occurred?

"Members themselves run the group, with leadership usually rotating among the members."

During an intake assessment, the nurse asks a client physiological and psychosocial questions. The client angrily responds, "I'm here for my heart problems, not for my head." Which is the nurse's best response?

"Psychological stress can affect medical conditions."

When asked to play a game, a client diagnosed with paranoid schizophrenia becomes agitated. The client states, "Do you want to be my girlfriend?" Which nursing response is best?

"Remember, we have a professional relationship. Are you feeling uncomfortable?"

Which statement demonstrates that the nurse is using a cognitive approach when teaching a client about panic disorder?

"Remind yourself that symptoms of a panic attack are time-limited and will end."

A client diagnosed with somatization disorder is most likely to exhibit which personality disorder characteristic?

"Self-dramatizing, attention seeking, overly gregarious, and seductive"

A geriatric nurse is teaching student nurses about the development of delirium in older adults. Which statement by the nurse is most accurate?

"Taking multiple medications may lead to adverse interactions or toxicity."

A client tells the nurse, "I experience stress on a regular basis. Why do I feel so terrible this time?" Which is the nurse's most appropriate response?

"Your reactions to past experiences influence your current feelings."

Common physical signs and symptoms of agitated delirium include: A. constricted pupils, drowsiness, and cool skin. B. hypertension, tachycardia, and diaphoresis. C. chest pain, shortness of breath, and bradycardia. D. confusion, hypotension, and pallor.

B. hypertension, tachycardia, and diaphoresis

Place the selected steps of the problem-solving process in the correct order. 1. Determine risks and benefits of each option. 2. Formulate goals to resolve the stressful situation. 3. Implement a second alternative. 4. Study the alternatives for dealing with the situation.

2, 4, 1, 3

Place Kubler-Ross' stages of feelings related to loss in the correct order. 1. Anger 2. Bargaining 3. Denial 4. Depression 5. Acceptance

3, 1, 2, 4, 5

When caring for a patient with a behavioral crisis, you should ensure your own safety and then: A. identify any life-threatening conditions. B. frisk the patient for dangerous weapons. C. determine if the patient is seeing a psychiatrist. D. apply oxygen with a nonrebreathing mask.

A. identify any life-threatening conditions

A clinician prescribes an additional medication for a client taking an antipsychotic agent. The medication is to be administered "prn for EPS." Which nursing assessment indicates the need to give this medication? A. WBCs <3000/mm3 B. Tremors and a shuffling gait C. Dry mouth D. Generalized seizures

??? - probably B

What is the "percentage rate of deterioration" per year? a) 1% b) 3% c) 10% d) 35%

A

Which therapeutic communication technique is being used in this nurse-client interaction? Client: "My father spanked me often." Nurse: "Your father was a harsh disciplinarian." a) Restatement b) Offering general leads c) Focusing d) Accepting

A Rationale: The nurse is using the therapeutic communication technique of restatement. Restatement involves repeating the main idea of what the client has said. The nurse uses this technique to communicate that the client's statement has been heard and understood. Offering general leads to the client encourages the client to continue. Focusing is when the nurse notices a single idea or even a single word and works especially well with a client who is moving rapidly from one thought to another. Focusing is not therapeutic with a client who is very anxious. Accepting conveys an attitude of reception and regard

A client is prescribed medication for anxiety. Which medication can the nurse anticipate will be prescribed a) Alprazolam (Xanax) b) Bupropion (Wellbutrin) c) Lithium carbonate (Lithobid) d) Methylphenidate (Ritalin)

A Rationale: Alprazolam is a benzodiazepine used in the treatment of anxiety Bupropion is an antidepressant, also used for smoking cessation, and reducing symptoms of ADHD (unlabeled use) without anxiolytic effects, lithium is a mood stabilizer used in the management of bipolar disorder, and methylphenidate is an amphetamine used in the management of ADHD

A physically and emotionally healthy client has just been fired. During a routine office visit, he tells the nurse, "Perhaps this was the best thing to happen. Maybe I'll consider pursuing an art degree." The nurse determines the client perceives the stressor of his job loss as: a) A challenge b) A threat c) Harm/loss. d) Irrelevant

A Rationale: The client's statement indicates he perceives the loss of his job as a challenge and an opportunity for growth.

Which should the nurse recognize as an example of systematized amnesia?

A client can relate family memories but has no recollection of a particular brother.

Which client should a nurse identify as a potential candidate for involuntary commitment?

A client verbalizing intent to commit suicide.

While assessing a patient with an apparent behavioral crisis, he tells you that he sees a pink elephant in the corner of the room. This is an example of a(n): A. visual hallucination. B. visual delusion. C. auditory hallucination. D. auditory delusion.

A. visual hallucination

Ritalin

ADHD

The parent of a child with attention-deficit/hyperactivity disorder (ADHD) asks a nurse why the child has lost 10 pounds in the past 2 months. Which is the most appropriate nursing response? A) "Medications used to treat people with ADHD often cause decreased appetite." B) "Hyperactivity causes excess physical activity and increased caloric expenditure." C) "Side effects of the medications used to treat people with ADHD include nausea and vomiting." D) "Loss of appetite is a symptom of ADHD."

A) "Medications used to treat people with ADHD often cause decreased appetite."

A nurse is caring for a 5-year-old child with Tourette syndrome. Which class of medications is effective in the treatment of Tourette syndrome? A) Neuroleptics B) Antimanics C) Tricyclic antidepressants D) MAOIs (monoamine oxidase inhibitors)

A) Neuroleptics

The mother of a 4-year-old child with autism reports that the child has begun head banging since she returned to full-time employment. The mother has had difficulty finding adequate and appropriate caregivers for her son. Which priority nursing diagnosis for the child should the nurse assign? A) Risk for self-mutilation B) Impaired social interaction C) Impaired verbal communication D) Complicated grieving

A) Risk for self-mutilation

8. A nurse should identify topiramate (Topamax) as the drug of choice for which of the following conditions? (Select all that apply.) A. Binge eating with obesity B. Bingeing and purging with a diagnosis of bulimia nervosa C. Weight loss with a diagnosis of anorexia nervosa D. Amenorrhea with a diagnosis of anorexia nervosa E. Emaciation with a diagnosis of bulimia nervosa

A, B

While reviewing laboratory results of a newly admitted client, the nurse discovers that the client's thyroidstimulating hormone (TSH) levels are 6.7 mIU/L. The nurse anticipates the client will exhibit which symptoms? Select all that apply. A. Depression B. Fatigue C. Increased libido D. Mania E. Hyperexcitability

A, B A. Elevated TSH levels are indicative of hypothyroidism. Symptoms include depression and fatigue. Memory impairment, decreased libido, and suicidal ideation can occur with chronic hypothyroidism. B. Elevated TSH levels are indicative of hypothyroidism. Symptoms include depression and fatigue. Memory impairment, decreased libido, and suicidal ideation can occur with chronic hypothyroidism. Normal TSH ~ 0.4-4.2 mIU/L

Which of the following is true about aggression? Select all that apply. a) It is goal directed b) It's aim is to do harm to a person or object c) It has a requisite of intent d) It energizes immobilize the body for self-defense

A, B, C

Which of the following are characteristics of mild anxiety? Select all that apply. a) Mild anxiety enhances learning. b) Mild anxiety increases motivation. c) Mild anxiety leads to confusion and loss of concentration. d) Mild anxiety prepares people for action. e) Mild anxiety requires problem-solving assistance

A, B, D Rationale: Mild anxiety sharpens the senses, increases motivation for productivity, increases perceptual field, and heightens awareness in the environment.

A nursing student is developing a study guide related to historical facts about suicide. Which of the following facts should the student include? (Select all that apply.) A. In the Middle Ages, suicide was viewed as a selfish and criminal act. B. During the Roman Empire, suicide was treated by incineration of the body. C. Suicide was an offense in ancient Greece, and a common site burial was denied. D. During the Renaissance, suicide was discussed and viewed more philosophically. E. Old Norse traditionally set a person who committed suicide adrift in the North Sea.

A, C, D

Which of the following statements should a nurse recognize as true about defense mechanisms? Select all that apply. A. They are employed when there is a threat to biological or psychological integrity. B. They are controlled by the id and deal with primal urges. C. They are used in an effort to relieve mild to moderate anxiety. D. They are protective devices for the superego. E. They are mechanisms that are characteristically self-deceptive.

A, C, E

Which of the following were attributed to mental illness prior to the influence of Middle Eastern countries? Select all that apply. A. Supernatural Forces B. Medical conditions C. Disequilibrium of humors D. Personality E. Demons

A, C, E

Which of the following are correlated with an individual's affiliation with a religious faith? Select all that apply. A. Decreased incidence of mental illness B. Improved social relationships C. Access to health care D. Increased chance of survival following serious illness E. Better physical and mental health

A, D, E

A nurse is working with a client who has recently been under a great deal of stress. Which nursing recommendations would be most helpful to assist the client to cope with stress? Select all that apply. A. "Enjoy a pet." B. "Spend time with a loved one." C. "Listen to music." D. "Focus on the stressors." E. "Journal your feelings."

A,B,C,E A. Pets have been shown to decrease stress. B. Healthy relationships have been shown to decrease stress. C. Music has been shown to decrease stress. D. This is incorrect. Focusing on the stressors is more likely to increase stress. E. Journaling feelings has been shown to decrease stress.

Which client statement alerts the nurse that the client may be responding maladaptively to stress? A. "Avoiding contact with others helps me cope." B. "I really enjoy journaling; it's my private time." C. "I signed up for a yoga class this week." D. "I made an appointment to meet with a therapist."

A. "Avoiding contact with others helps me cope."

5. A nursing instructor is teaching students about the differences between the symptoms of anorexia nervosa and the symptoms of bulimia nervosa. Which student statement indicates that learning has occurred? A. "Clients diagnosed with anorexia nervosa experience extreme nutritional deficits, whereas clients diagnosed with bulimia nervosa do not." B. "Clients diagnosed with bulimia nervosa experience amenorrhea, whereas clients diagnosed with anorexia nervosa do not." C. "Clients diagnosed with bulimia nervosa experience hypotension, edema, and lanugo, whereas clients diagnosed with anorexia nervosa do not." D. "Clients diagnosed with anorexia nervosa have eroded tooth enamel, whereas clients diagnosed with bulimia nervosa do not."

A. "Clients diagnosed with anorexia nervosa experience extreme nutritional deficits, whereas clients diagnosed with bulimia nervosa do not."

A nurse is working with a client who has recently been under a great deal of stress. Which nursing recommendations would be most helpful to assist the client to cope with stress? Select all that apply. A. "Enjoy a pet." B. "Spend time with a loved one." C. "Listen to music." D. "Focus on the stressors." E. "Journal your feelings."

A. "Enjoy a pet." B. "Spend time with a loved one." C. "Listen to music." D. "Focus on the stressors."

A depressed client states, "I have a chemical imbalance in my brain. I have no control over my behavior. Medications are my only hope to feel normal again." Which response by the nurse is best? A. "Medications are one way to address chemical imbalances. Environmental and interpersonal factors can also have an impact on biological factors." B. "Biological factors are the sole cause of depression, so medications will improve your mood." C. "Environmental factors have been shown to exert the most influence in the development of depression." D. "Researchers have been unable to demonstrate a link between biology and genetics."

A. "Medications are one way to address chemical imbalances. Environmental and interpersonal factors can also have an impact on biological factors."

A mental health technician asks the nurse, "How do psychiatrists determine which diagnosis to give a patient?" Which is the nurse's best response? A. "Psychiatrists use criteria from the APA's Diagnostic and Statistical Manual of Mental Disorders." B. "Psychiatrists are required to follow hospital policy to diagnose mental disorders." C. "Psychiatrists make diagnoses based on the patient's behavior and other factors." D. "Psychiatrists use on of the 10 diagnostic labels from the American Medical Association."

A. "Psychiatrists use criteria from the APA's Diagnostic and Statistical Manual of Mental Disorders."

A client diagnosed with chronic migraine headaches is considering acupuncture. The client asks a clinic nurse, "How does this treatment work?" Which is the best response by the nurse? A. "Western medicine believes that acupuncture stimulates the body's release of pain-fighting chemicals called endorphins." B. "I'm not sure why he suggested acupuncture. There are a lot of risks, including HIV." C. "Acupuncture works by encouraging the body to increase its development of serotonin and norepinephrine." D. "Your acupuncturist is your best resource for answering your specific questions."

A. "Western medicine believes that acupuncture stimulates the body's release of pain-fighting chemicals called endorphins."

A nurse is interviewing a distressed client who reports being fired after 15 years of loyal employment. Which of the following questions best assists the nurse to determine the client's appraisal of the situation? Select all that apply. A. "What resources have you used previously in stressful situations?" B. "Have you ever experienced a similar stressful situation?" C. "Who do you think is to blame for this situa- tion?" D. "Why do you think you were fired from your job?" E. "What skills do you possess that might lead to gainful employment?"

A. "What resources have you used previously in stressful situations?" B. "Have you ever experienced a similar stressful situation?" E. "What skills do you possess that might lead to gainful employment?"

Which of the following situations is NOT an example of a patient with a mental illness? A. A 55-year-old male who experiences a panic attack after being diagnosed with cancer B. A 41-year-old male with sadness and despair with no appreciable underlying cause C. A 38-year-old female with depression and anxiety for at least one month's duration D. A 29-year-old female whose violent actions are no longer tolerated by society

A. A 55-year-old male who experiences a panic attack after being diagnosed with cancer

Which of the following emotionally disturbed patients could legally refuse EMS treatment and transport? A. A 77-year-old male who is severely depressed over the death of his wife B. A 55-year-old confused male with auditory and visual hallucinations C. A 60-year-old female with a history of schizophrenia and bizarre behavior D. A 43-year-old female who is mumbling incomprehensible words

A. A 77-year-old male who is severely depressed over the death of his wife

Meditation has been shown to be an effective stress management technique. Which nursing assessment indicates meditation has been effective? A. An achieved state of relaxation B. An achieved insight into one's feelings C. A demonstration of appropriate role behav- iors D. An enhanced ability to problem-solve

A. An achieved state of relaxation

The nurse is caring for a client admitted to the palliative care unit. The client's spouse has been at the client's bedside since the client was admitted. One week ago, the spouse began to visit 2 or 3 hours a day. The nurse understands the spouse is experiencing which of the following? A. Anticipatory grief B. Bereavement C. Depression D. Resolution

A. Anticipatory grief

Which concepts are included in Hobfoll's Conservation of Resources theory? Select all that apply. A. Availability of resources B. Disequilibrium C. Genetics D. Past experiences E. Resilience

A. Availability of resources C. Genetics D. Past experiences

When an individual's stress response is sustained over a long period, the nurse anticipates which physiological effect? A. Decreased resistance to disease B. Increased libido C. Decreased blood pressure D. Increased inflammatory response

A. Decreased resistance to disease

Place Kübler-Ross' stages of feelings related to loss in the correct order. 1. _____Anger 2. _____Bargaining 3. _____Denial 4. _____Depression 5. _____Acceptance

ANS: 3, 1, 2, 4, 5 The five stages of feelings and behaviors individuals experience in response to real, perceived, or anticipated loss are: - Denial - Anger - Bargaining - Depression - Acceptance

A new psychiatric nurse states, "This client's use of defense mechanisms should be eliminated." Which is a correct evaluation of the nurse's statement? A. Defense mechanisms are self-protective responses to stress and do not need to be eliminated B. Defense mechanisms are maladaptive attempts of the ego to manage anxiety and should always be eliminated C. Defense mechanisms are used by individuals with weak ego integrity and should not be eliminated D. Defense mechanisms cause disintegration of the ego and should be fostered and encouraged

A. Defense mechanisms are self-protective responses to stress and do not need to be eliminated

A nurse is assessing a client who appears to be experiencing moderate anxiety during questioning. Which symptoms might the client demonstrate? Select all that apply. A. Fidgeting B. Laughing inappropriately C. Palpitations D. Nail biting E. Extremely limited attention span

A. Fidgeting B. Laughing inappropriately D. Nail biting

Most cultures label behavior as mental illness on the basis of which of the following criteria? A. Incomprehensibility and cultural relativity B. Strength of character and ethics C. Goal directedness and high energy D. Creativity and good coping skills

A. Incomprehensibility and cultural relativity

How is the DSM-5 useful in the practice of psychiatric nursing? Select all that apply. A. It informs the nurse of accurate and reliable medical diagnosis. B. It represents progress toward a more holistic view of mind-body. C. It provides a framework for interdisciplinary communication. D. It provides a template for nursing care plans. E. It provides a framework for communication with the client.

A. It informs the nurse of accurate and reliable medical diagnosis. B. It represents progress toward a more holistic view of mind-body. C. It provides a framework for interdisciplinary communication.

Which of the following findings does NOT indicate the potential for violence? A. Large physical body size B. Agitated or depressed C. Uncoordinated movements D. Withdrawn or detached

A. Large physical body size

Which of the following are cultural aspects of mental illness? Select all that apply. A. Local or cultural norms define pathological behavior. B. The higher the social class the greater the recognition of mental illness behaviors. C. Psychiatrists typically see patients when the family can no longer deny the illness. D. The greater the cultural distance from the mainstream of society, the greater the likelihood that the illness will be treated with sensitivity and compassion.

A. Local or cultural norms define pathological behavior. B. The higher the social class the greater the recognition of mental illness behaviors. C. Psychiatrists typically see patients when the family can no longer deny the illness.

A nurse is assessing a client diagnosed with Schizophrenia for the presence of hallucinations. Which therapeutic communication technique used by the nurse is an example of making observations? A. "You appear to be talking to someone I do not see." B. "Please describe what you are seeing." C. "Why do you continually look in the corner of this room?" D. "If you hum a tune, the voices may not be so distracting."

A. Making observations involves verbalizing what is observed or percieved. This encourages the client to recognize specific behaviors and make comparisons with the nurse's perceptions.

A mental health nurse is speaking with parents who are concerned about their 15-year-old identical twins' responses to stress. One twin becomes anxious and irritable, while the other withdraws and cries. Which is the nurse's best response? A. Reactions to stress are relative rather than absolute. Individual responses to stress vary. B. It is abnormal for identical twins to react differently to similar stressors. C. Identical twins should share the same temperament and respond similarly to stress. D. Environmental influences to stress weigh more heavily than genetic influences.

A. Reactions to stress are relative rather than absolute. Individual responses to stress vary.

A client with depression and substance abuse has an interrupted sleep pattern. She demands that her psychiatrist prescribe her a sedative. Which teaching should the nurse provide about the rationale for the use of nonpharmacological interventions instead? A. "Sedative-hypnotics are potentially addictive and gradually lose their effectiveness as one builds up tolerance to them." B. "Sedative-hypnotics work best in combination with other techniques." C. "Sedative-hypnotics are not permitted for use in patients with substance abuse disorders." D. "Sedative-hypnotics are not as effective as the antidepressant medications for treating sleep disturbances."

A. Sedative-hypnotics are potentially addictive and should be used with caution by clients with a history of substance use disorders, as tolerance can easily develop.

Which psychoneurotic responses to severe anxiety are identified in the DSM-5? A. Somatic symptom disorders B. Grief reactions C. Psychosis D. Bipolar disorder

A. Somatic symptom disorders

A nurse observes dental deterioration when assessing a client diagnosed with bulimia nervosa. What explains this assessment finding? A. The emesis produced during purging is acidic and corrodes the tooth enamel. B. Purging causes the depletion of dietary calcium. C. Food is rapidly ingested without proper mastication. D. Poor dental and oral hygiene leads to dental caries.

A. The emesis produced during purging is acidic and corrodes the tooth enamel.

During the first interview with a man from Syria who has just lost his son in a car accident, the female nurse hugs him. Which is accurate regarding the nurse's action? A. It is unacceptable because it may pose a breech of cultural norms. B. It demonstrates empathy and encourages expression of feelings. C. It is the therapeutic technique of offering self. D. It is an inappropriate action due to poor timing.

A. The nurse's action is unacceptable because it imposes a breach of cultural norms. During communication, some Arab Americans stand close together, maintain steady eye contact, and may touch the other's hand or shoulder but only between members of the same sex.

Joey, age 8 years, takes methylphenidate (Ritalin) for attention-deficit/hyperactivity disorder (ADHD). His mother complains to the nurse that Joey has a very poor appetite, and she struggles to help him gain weight. Which teaching will the nurse provide? A. Administer Joey's medication immediately after meals. B. Administer Joey's medication at bedtime. C. Skip a dose of the medication when Joey does not eat anything. D. Assure Joey's mother that Joey will eat when he is hungry.

A. This is correct. Anorexia and weight loss are side effects of stimulant medications such as methylphenidate (Ritalin). Stimulants should be administered immediately after meals and weight should be monitored at least once a week. B. This is incorrect. Stimulants can cause insomnia. C. This is incorrect. Regular dosing needs to be maintained D. This is incorrect. Anorexia and weight loss are side effects of stimulant medications such as methylphenidate (Ritalin). Allowing weight loss and decreased intake will have effects on physical and mental performance.

Which client statement alerts the nurse that the client may be responding maladaptively to stress? A. "Avoiding contact with others helps me cope." B. "I really enjoy journaling; it's my private time." C. "I signed up for a yoga class this week." D. "I made an appointment to meet with a therapist."

A. This is correct. Reliance on social isolation as a coping mechanism is a maladaptive method to relieve stress. It can prevent learning appropriate coping skills and can prevent access to needed support systems. B. This is incorrect. Journaling is not considered a maladaptive coping method. C. This is incorrect. Group exercise is not considered a maladaptive coping method. D. This is incorrect. Seeing a therapist is not considered a maladaptive coping method.

A mental health nurse is speaking with parents who are concerned about their 15-year-old identical twins' responses to stress. One twin becomes anxious and irritable, while the other withdraws and cries. Which is the nurse's best response? A. "Individual responses to stress are affected by many factors and can vary." B. "Identical twins should not react differently when experiencing stress." C. "Identical twins should have similar dispositions and responses to stress." D. "Environmental factors influence reactions to stress more than genetic factors."

A. This is correct. Responses to stress are variable among individuals and may be influenced by perception, past experiences, and environmental factors in addition to genetic factors. B. This is incorrect. Each twin will have unique responses to stress. C. This is incorrect. The twins' dispositions will reflect their individual personalities. D. This is incorrect. In addition to genetic factors, responses to stress can be influenced by perception, past experiences, and environmental factors.

A student nurse is learning about the appropriate use of touch when communicating with clients diagnosed with psychiatric disorders. Which statement by the instructor best provides information about this aspect of therapeutic communication? A. "Touch carries a different meaning for different individuals." B. "Touch is often used when deescalating volatile client situations." C. "Touch is used to convey interest and warmth." D. "Touch is best combined with empathy when dealing with anxious clients."

A. Touch can elicit both negative and posivite reactions, depending on the people involved and the circumstances of the interaction.

During your assessment of a 43-year-old male with suicidal thoughts, he becomes agitated and appears uncomfortable. You should: A. continue talking to him while your partner requests police backup. B. immediately assess his blood glucose level to rule out hypoglycemia. C. immediately restrain him to prevent him from injuring himself. D. call for a paramedic unit so they can administer a sedative to him.

A. continue talking to him while your partner requests police backup

The length and severity of an acute psychotic episode caused by a mind-altering substance depends on the: A. duration of the substance being metabolized within the body. B. patient's vital signs at the time the substance was introduced. C. AEMT's ability to effectively communicate with the patient. D. patient's gender, age, and socioeconomic background.

A. duration of the substance being metabolized within the body

To minimize the risk of legal implications when managing an emotionally disturbed 33-year-old male, you should: A. gain his confidence so that he consents to care. B. allow him to refuse transport if he appears alert. C. transport him to a specialized psychiatric facility. D. restrain him to prevent him from injuring himself.

A. gain his confidence so that he consents to care

9. A patient received one dose of flumazenil (Romazicon). What is the nurse's next action? a. Carefully observe for preflumazenil symptoms. b. Teach the patient about dietary restrictions. c. Prevent injury during seizure activity. d. Force 500 ml oral fluids over 2 hours.

ANS: A Flumazenil, which is given to patients who have overdosed with benzodiazepines, is a benzodiazepine receptor antagonist. Response occurs within 30 to 60 seconds; however, it might not reverse associated respiratory depression. Because it has a short duration of action and does not speed metabolism of benzodiazepines, the nurse must be vigilant for signs that the patient is reverting to the preflumazenil state. Administration of flumazenil might need to be repeated several times.

23. Which individual would be most likely to experience a paradoxical reaction to a benzodiazepine drug? a. A child with attention-deficit/hyperactivity disorder. b. An adult with obsessive-compulsive disorder. c. A teenager with an eating disorder. d. An adult with major depression.

ANS: A Paradoxical reactions to benzodiazepines are most likely in children, older adults, and persons with poor impulse control (such as ADHD) or organic brain syndromes.

14. A patient diagnosed with social phobia begins propranolol (Inderal). The nurse should teach the patient to expect: a. that sympathetic nervous system symptoms of anxiety will be reduced. b. to experience a sense of euphoria for 30 minutes after taking the drug. c. to have amnesia for the social situations that are most intimidating. d. to feel a little drowsy but have no orthostatic hypotension.

ANS: A Propranolol is a beta blocker that interrupts the physiologic responses of anxiety associated with social phobias, such as sweaty palms. Bradycardia may be associated with lightheadedness. The other options are not likely.

18. A patient had 5 emergency room visits in the past month and reports, "I feel so nervous. I think I'm having heart attacks." The patient is diagnosed with panic attacks. Which comment by the nurse shows understanding of treatment for panic attacks? a. "SSRI antidepressants are often helpful for long-term treatment and prevention of panic attacks." b. "Benzodiazepine tranquilizers are therapeutic for long-term treatment and prevention of panic attacks." c. "No medications are particularly helpful for panic attacks. Let's work on some strategies to help you manage your fears." d. "Panic attacks result from an instability of the neurotransmitter acetylcholine. Meditation will be more helpful than drugs."

ANS: A Selective serotonin reuptake inhibitors (SSRIs) are approved for panic disorder and might be the most effective and safest agents for prophylaxis and long-term treatment. Benzodiazepines are effective during a panic attack but should not be used for long-term treatment because of the abuse and dependence potentials. It's important that the nurse show compassion for the patient's distress. Meditation may help overall, but not during panic attacks.

6. A patient has taken clonazepam (Klonopin) for years to manage panic attacks but impulsively stopped the drug. Thirty hours later, the patient comes to the emergency room in distress. What is the nurse's priority action? a. Begin seizure precautions. b. Refer the patient for addiction counseling. c. Institute a behavior modification program. d. Prepare to administer flumazenil (Romazicon)

ANS: A There is evidence to suggest that abrupt withdrawal of clonazepam might precipitate status epilepticus. With this in mind, withdrawal from long-term use warrants seizure precautions. The patient does not have an overdose, so flumazenil (Romazicon) is not indicated. The other options are inappropriate.

21. A patient who has been taking a benzodiazepine for panic attacks is to be started on buspirone (BuSpar). Which instruction should the nurse provide? a. "Take decreasing doses of the benzodiazepine for several days until the buspirone becomes effective." b. "Stop taking the benzodiazepines immediately. Wait 2 days, and then start the buspirone." c. "You should take buspirone only once a day. More frequent dosing can cause dependency." d. "Tolerance to buspirone may develop in about a month, requiring larger doses to be prescribed."

ANS: A Two factors suggest that the patient should take tapering doses of benzodiazepine while beginning buspirone therapy. Benzodiazepines should be tapered gradually for discontinuation to avoid withdrawal. Buspirone takes 7 to 10 days to begin to exert its therapeutic effect. The other statements about buspirone are incorrect.

11. A patient in the emergency room has status epilepticus. The nurse should anticipate administration of: a. diazepam (Valium). b. buspirone (BuSpar). c. clorazepate (Tranxene). d. chlordiazepoxide (Librium).

ANS: A Valium is the drug of choice in status epilepticus because of its rapid action. Each of the other benzodiazepines has a slower onset of action. Buspirone is not indicated to treat seizures.

2. A patient takes antacids, cimetidine (Tagamet), and phenytoin (Dilantin). The health care provider prescribes a benzodiazepine for anxiety. Which drug interactions would the nurse anticipate? Select all that apply. a. Increased plasma level of benzodiazepine related to cimetidine therapy b. Increased absorption of the benzodiazepine if taken with the antacid c. Euphoria and disinhibition associated with phenytoin therapy d. Serotonin syndrome associated with cimetidine use e. Potential phenytoin toxicity

ANS: A, E Cimetidine increases the plasma level of benzodiazepines. The benzodiazepine interferes with phenytoin metabolism, thus increasing serum levels of the anticonvulsants. The distracters do not reflect actual interactions.

17. A patient took a benzodiazepine for 4 weeks but will now change to buspirone (BuSpar) for long-term treatment of anxiety. The benzodiazepine is tapered off as the buspirone is begun. Important information the patient should receive about buspirone is that it: a. produces profound sedation. b. will be effective in 7 to 10 days. c. has a high risk for development of dependence. d. is often associated with cross-tolerance with other CNS depressants.

ANS: B Buspirone provides anxiety relief within 7 to 10 days from the time it is begun. For this reason, benzodiazepines are continued for their anxiolytic effect and gradually tapered as the buspirone becomes effective. The other options are incorrect.

13. A patient seeking treatment for anxiety says, "I can't think. My job depends on my ability to think. I need medicine, but the drugs I took a few years ago made me too sleepy. I could lose my job." What information is most important for the nurse to consider when formulating a response? a. All antianxiety medication has sedating properties. b. Buspirone (BuSpar) alleviates anxiety without sedation or cognitive clouding. c. The patient's description of anxiety does not warrant treatment with medication. d. The patient may be trying to manipulate the nurse to assist with getting the desired prescription.

ANS: B Buspirone's action is entirely different from that of the benzodiazepines. It reduces anxiety, with its accompanying concentration and cognitive problems, but without CNS depression. The patient's description of anxiety indicates that it is interfering with daily life, so medication may be helpful. There is no evidence that the patient is trying to manipulate the nurse.

16. A patient is to receive lorazepam (Ativan) 1 mg PO t.i.d. for 4 days. The nurse understands that this drug was prescribed for a short period because: a. hypoglycemic reactions occur with continued use. b. therapeutic dose dependence develops quickly. c. neutropenia is likely with long-term use. d. glaucoma results from long-term therapy.

ANS: B Dependence to benzodiazepines develops rather quickly. Even when taking prescribed doses, individuals develop a psychological need for the drug. Prescribing for only several days circumvents this problem. The other options are incorrect.

8. The nurse would expect to administer flumazenil (Romazicon) for a patient: a.in acute alcohol withdrawal. b.with a benzodiazepine overdose. c.with benzodiazepine-resistant anxiety. d.with anxiety associated with a psychotic disorder.

ANS: B Flumazenil is a benzodiazepine receptor antagonist. Response occurs within 30 to 60 seconds; however, it might not reverse associated respiratory depression. Because it has a short duration of action and does not speed metabolism of benzodiazepines, administration of flumazenil might need to be repeated several times. Flumazenil is not indicated for treatment of any of the other conditions.

20. A health care provider prescribes lorazepam (Ativan) for an anxious older adult. What is the nurse's best action? a.Assess for a history of drug abuse. b.Administer the drug as prescribed. c.Confer with the health care provider. d.Assess the patient's pupillary reaction to light.

ANS: B Lorazepam is a benzodiazepine that has a short half-life. It might be administered safely to older adult patients, although the dose should often be modified downward. It is inadvisable to give benzodiazepines with longer half-lives to older adult patients.

22. Singer Michael Jackson allegedly abused alprazolam (Xanax) and propofol (Diprivan), an anesthetic. Which inference applies? a. The singer had a substance dependence problem, but these medications had no interaction. b. Potentially lethal sedation and CNS depression would be expected with this drug combination. c. Tolerance to propofol probably developed very quickly in the presence of alprazolam. d. This drug combination was safe, but the singer needed closer medical supervision.

ANS: B Propofol, an anesthetic, would have a predictable additive effect with alprazolam in producing significant sedation and CNS depression. The singer likely needed closer medical supervision, but one cannot state that the combination of drugs was safe. Tolerance to alprazolam would occur regardless of use of propofol.

A patient started diazepam (Valium) 5 mg b.i.d. 6 months ago. Now, the patient requires 10 mg to achieve the same effect. This phenomenon results from: a. addiction. b. tolerance. c. dependence. d. disinhibition.

ANS: B Tolerance is the need for increasing amounts of a substance to achieve the same effects. The other terms, defined in the text, do not account for this phenomenon

3. A patient in the emergency room is suspected to have an overdose of benzodiazepines. Which assessment findings validate this diagnosis? Select all that apply. a. Blood pressure 180/94 b. Diminished reflexes c. Hypervigilance d. Somnolence e. Confusion

ANS: B, D, E Benzodiazepine toxicity may result from an overdose. Assessment findings include hypotension, somnolence, confusion, and diminished reflexes.

1. The teaching plan for a patient beginning oxazepam (Serax) should include instructions to: Select all that apply. a. take the drug on an empty stomach. b. avoid discontinuing the drug abruptly. c. stop taking the drug if side effects occur. d. drink only moderate amounts of alcohol. e. avoid herbal preparations.

ANS: B, E Patients must be informed that abrupt discontinuation of benzodiazepines produces withdrawal symptoms. Use of herbal preparations such as kava-kava and valerian can produce harmful additive effects. The other options contain information that is inappropriate to teach patients.

10. The nurse would expect a patient with which comorbid diagnosis to have a magnified response to the usual dose of a benzodiazepine drug? a. Rheumatoid arthritis b. Migraine headache c. Hepatic cirrhosis d. Osteoporosis

ANS: C Benzodiazepines are metabolized in the liver. The cirrhotic liver will slow the metabolism rate of the drugs, leading to an exaggerated response. The distracters are not associated with decreased hepatic function.

7. Which patient has the greatest risk for overdose with a benzodiazepine? A patient who: a. takes the drug with antacids b. takes the drug before meals c. combines the drug with alcohol d. experiences depression as well as anxiety

ANS: C Benzodiazepines taken with alcohol produce marked CNS depression, even death. Antacids prevent absorption. Larger doses of benzodiazepines by themselves are rarely lethal. Depression in and of itself is not an indicator of overdose risk. Suicidal ideation might be present, but benzodiazepines by themselves are rarely lethal.

The teaching plan for a patient beginning buspirone (BuSpar) should include information identifying this drug as a: a. norepinephrine inhibitor. b. serotonergic antagonist. c. serotonin agonist. d. GABA inhibitor.

ANS: C It is believed that buspirone is a serotonin agonist. Because buspirone is not a benzodiazepine, it does not bind to benzodiazepine receptor sites, affect GABA, or affect norepinephrine. This accounts for its different effects and lack of CNS depression as side effects.

3. A patient with agoraphobia took alprazolam (Xanax) 0.5 mg t.i.d. for 3 months and then discontinued it. The next day, the patient called the nurse complaining of insomnia, shakiness, and sweating. The nurse's assessment questions should focus on whether the patient: a. may have also been drinking alcohol or taking antihistamines. b. has built up tolerance to alprazolam and needs an increased dose. c. is having withdrawal symptoms from abrupt discontinuation of the drug. d. has progressed to panic attacks and needs a nonbenzodiazepine medication.

ANS: C The patient's symptoms suggest benzodiazepine withdrawal. The nurse knows that patients often attempt to manage their own care by discontinuing medication when they begin to feel better. Benzodiazepines should be slowly withdrawn if withdrawal symptoms are to be avoided. Drinking alcohol would result in different symptoms. Development of tolerance and panic attack symptoms would be different from those mentioned.

5. A patient has taken diazepam (Valium) for one week for back spasms. The patient complains of "feeling sleepy all the time." The nurse should tell the patient: a."The dosage probably needs to be decreased." b."Drowsiness indicates a paradoxical reaction to the drug." c."Tolerance to the sedative effect of the drug will develop quickly." d."Sleepiness is an unavoidable side effect of nonbenzodiazepine drugs."

ANS: C Tolerance to most side effects of benzodiazepines, including drowsiness, develops quickly. There is no need to decrease the dosage. Drowsiness is an expected reaction, not a paradoxical one. Valium is a benzodiazepine.

19. The half-life of a benzodiazepine drug is 20 hours for a young adult, but for an older adult it is likely to be: a.10 hours. b.30 hours. c.40 hours. d. 80 hours.

ANS: D Because of decreased liver size and function in older adults, the half-life of benzodiazepines is markedly lengthened. Benzodiazepines with long half-lives are unsuitable for older adults.

Lorazepam (Ativan) reduces anxiety by: a. increasing serotonin levels. b. blocking dopamine receptors. c. depressing norepinephrine levels. d. potentiating gamma-aminobutyric acid (GABA).

ANS: D Benzodiazepines enhance the effects of the inhibitory neurotransmitter GABA, slowing neuronal firing. They do not affect dopamine, serotonin, or norepinephrine.

An emergency room patient was very anxious after a serious car accident. Lorazepam (Ativan) 2 mg IM was administered. One hour later, which finding indicates to the nurse that the medication was effective? a. Impaired problem-solving skills b. Increased alertness and attention c. Increased verbalization and activity d. Reduced agitation and environmental scanning

ANS: D Benzodiazepines mute incoming stimuli and evoke less reaction. The hyperalertness and environmental scanning that accompany high anxiety are notably decreased when the drug is effective. Impaired problem solving is a negative outcome. Because of its sedating properties, the individual might not be more alert, talkative, or active.

15. A patient complains, "I have the same thoughts over and over. I feel compelled to count all my footsteps." The nurse can expect the health care provider to prescribe: a. alprazolam (Xanax). b. propranolol (Inderal). c. clonazepam (Klonopin). d. clomipramine (Anafranil).

ANS: D Clomipramine is an antidepressant that has proven effective for obsessive-compulsive disorder (OCD). The other drugs have no proven effectiveness in treating OCD.

The nursing assessment of a child with separation anxiety disorder is most likely to reveal which factor? A) The child has a history of antisocial behaviors. B) The mother has an anxiety disorder. C) The child previously had an extroverted temperament. D) The parents have an inconsistent parenting style.

B) The mother has an anxiety disorder.

A client diagnosed with dementia is exhibiting behavioral problems on a daily basis. At change of shift, the client's behavior escalates from pacing to screaming and flailing. Which action should be a nursing priority?

Assess environmental triggers and potential unmet needs

Which statement is most accurate regarding the assessment of clients diagnosed with psychiatric problems?

Assessment provides a holistic view of the client, including biopsychosocial aspects.

Which disorder does the nurse recognize as a disorder in the DSM-5? A. Morbid obesity B. Generalized anxiety disorder C. Essential hypertension D. Bereavement

B. Generalized anxiety disorder

A client diagnosed with Alzheimer's dementia has impairments of memory and judgment and is incapable of performing activities of daily living. Which nursing intervention should take priority?

Assist with bathing and toileting

At what time during a 24-hour period should a nurse expect clients diagnosed with Alzheimer's to exhibit more pronounced symptoms of dementia?

At twilight

A client diagnosed with Posttraumatic Stress Disorder is admitted to an inpatient psychiatric unit for evaluation and medication stabilization. Which therapeutic communication technique used by the nurse is an example of a broad opening? A. "What occurred prior to the rape, and when did you go to the emergency department?" B. "What would you like to talk about?" C. "I notice you seem uncomfortable discussing this." D. "How can we help you feel safe during your stay here?"

B

The hallmark presentation of Alzheimer's disease in a younger patient is: a) memory impairment b) spatial disorientation c) loss of smell or taste d) partial complex seizures

B

The patient with the following disorder has a major risk of developing Parkinson's disease within 11 years: a) sleep apnea b) REM sleep disorder c) night terrors d) insomnia

B

A client tells the nurse, "I experience stress on a regular basis. Why do I feel this way?" Which is the nurse's most appropriate response? a) "Genetics has nothing to do with your temperament or feelings." b) "Your reactions to past experiences influence your current feelings." c) "Your stress level is lower when you are in good physical health." d) "You'll experience less stress if you use appropriate coping skills."

B Rationale: Past experiences are occurrences that result in learned patterns that can influence an individual's current adaptation response. They include previous exposure to the stressor or other stressors in general, learned coping responses, and degree of adaptation to previous stressors.

A student nurse has just entered a psychiatric rotation. The student asks a nursing instructor, "How will we know if someone may get violent?" Which is the most appropriate reply by the nursing instructor? a) You can't really say for sure. There are limited indicators of potential violence." b) "Certain behaviors indicate a potential for violence. They are labeled as a 'prodromal syndrome' and include rigid posture, clenched fists, and raised voice." c) "Any client can become violent, so it is best to be aware of your surroundings at all times." d) "When a client suddenly becomes quiet, is withdrawn, and maintains a flat affect, this is an indicator of potential violence."

B Rationale: Rigid posture, clenched fists, and raised voice are predictors of violent behavior - the prodromal syndrome.

A client was just informed of cognitive test results that indicate her daughter has mild mental retardation (MR). The client questions what this means. Which is the most appropriate nursing response regarding mild MR? A) "Children with mild MR need constant supervision." B) "Children with mild MR develop academic skills up to a sixth-grade level." C) "Children with mild MR appear different from their peers." D) "Children with mild MR have significant sensory-motor impairment."

B) "Children with mild MR develop academic skills up to a sixth-grade level."

The mother of a child with a new diagnosis of an autistic disorder has come to an emergency department of a children's hospital after an episode of head banging by her son. She is sobbing as a nurse enters the room. Upon inquiring, the mother cries, "I'm such a terrible mother. What did I do to cause this behavior in my son?" Which is the most appropriate nursing response concerning the cause of autism? A) "Researchers really don't know what causes autism." B) "Poor parenting doesn't cause autism. Research has shown that abnormalities in brain structure or function that are beyond your control are to blame." C) "The mother appears to play a greater role in the development of the disorder than the father." D) "Lack of early infant bonding with the mother may be a cause of autism. Did you breast-feed or bottle-feed?"

B) "Poor parenting doesn't cause autism. Research has shown that abnormalities in brain structure or function that are beyond your control are to blame."

An instructor is teaching nursing students about neurotransmitters. Which term best explains the process by which neurotransmitters released into the synaptic cleft return to the presynaptic neuron? A. Regeneration B. Reuptake C. Recycling D. Retransmission

B. Reuptake

Which of the following nursing statements demonstrate useful feedback? Select all that apply. A. "Hitting the wall yesterday was not the best choice to express your anger." B. "The hospital has a support group on Tuesdays for those who want to quit smoking." C. "It appears you want to sit near the nurse's station when the morning meeting starts." D. "Your behavior has been unacceptable since you were admitted to this unit." E. "I noticed you participated in group more this afternoon than this morning."

B, C, E B. Feedback is useful when it encourages self-sufficiency and imparts information rather than offering advice. C. Useful feedback should be directed toward behavior the client has the capacity to modify. In this situation, the nurse can explore the reason the client sits near the nurse's station and assist the client to modify the behavior when appropriate. E. Useful feedback should be directed toward behavior that the client has the capacity to modify and is specific. This statement is directed toward specific behavior the patient can modify (participating in group).

Prior to interviewing a client of a different culture, which of the following questions should the nurse consider asking? Select all that apply. A. Would using perfume products be acceptable? B. Who may be expected to be present during the client interview? C. Should communication patterns be modified to accommodate this client? D. How much eye contact should be made with the client? E. Would hand shaking be acceptable?

B,C,D,E The nurse should consider who might be with the client during the interview, modifications of comm. patterns, amount of eye contact, and hand-shaking acceptability.

The nurse recognizes which principle underlies effective patient teaching? A. Moderate to severe anxiety increases patient learning B. Mild anxiety enhances patient learning C. Panic-level anxiety improves nurses' teaching D. Severe anxiety intensifies concentration and enhances attention

B. Mild anxiety enhances patient learning

Which nursing statement about the concept of neuroses is most accurate? A. "An individual experiencing neurosis is unaware that he or she is experiencing distress." B. "An individual experiencing neurosis feels helpless to change his or her situation." C. "An individual experiencing neurosis is aware of psychological causes of his or her behavior." D. "An individual experiencing neurosis has a loss of contact with reality."

B. "An individual experiencing neurosis feels helpless to change his or her situation."

Teaching regarding the concepts of mental health and mental illness is effective when the student nurse states which of the following? A. "The concepts are rigid and based in religious beliefs" B. "The concepts are multidimensional and culturally defined" C. "The concepts are universal and unchanging" D. "The concepts are fixed and unidimensional"

B. "The concepts are multidimensional and culturally defined"

A client tells the nurse, "I experience stress on a regular basis. Why do I feel this way?" Which is the nurse's most appropriate response? A. "Genetics has nothing to do with your tem- perament or feelings." B. "Your reactions to past experiences influence your current feelings." C. "Your stress level is lower when you are in good physical health." D. "You'll experience less stress if you use appropriate coping skills."

B. "Your reactions to past experiences influence your current feelings."

According to Maslow's hierarchy of needs, which situation on an inpatient psychiatric unit would require priority intervention by a nurse? A. A client rudely complaining about limited visiting hours B. A client exhibiting aggressive behavior toward another client C. A client stating that no one cares D. A client verbalizing feelings of failure

B. A client exhibiting aggressive behavior toward another client

6. A client who is 5 foot 6 inches tall and weighs 98 pounds is admitted with a medical diagnosis of anorexia nervosa. Which nursing diagnosis would take priority at this time? A. Ineffective coping R/T food obsession B. Altered nutrition: less than body requirements R/T inadequate food intake C. Risk for injury R/T suicidal tendencies D. Altered body image R/T perceived obesity

B. Altered nutrition: less than body requirements R/T inadequate food intake

When used in combination with anxiolytic medication, alcohol leads to ____ effects and caffeine leads to ____ effects. A. Increased; increased B. Increased; decreased C. Decreased; decreased D. Decreased; increased

B. Anxiolytic medications work by depressing certain CNS functions. Alcohol is also a CNS depressant that potentiates (increases) effects of anxiolytics. Caffeine is a CNS stimulant and inhibits (decreases) their effects.

A nurse is providing education on anxiety and stress management. Which of the following should be identified as the most important initial step in learning how to manage anxiety? A. Diagnostic blood tests B. Awareness of factors creating stress C. Relaxation exercises D. Identification of support systems

B. Awareness of factors creating stress

Which of the following would be the MOST important parameter(s) to assess in a patient with a behavioral emergency? A. Pupillary response B. Blood glucose level C. Baseline vital signs D. Glasgow Coma Score

B. Blood glucose level

Which indicates to the nurse a client is at risk for developing a mental disorder? A. Expresses thoughts, feelings, and behaviors included amond Diagnostic and Statistical Manual of Mental Disorders, Fifth Edition (DSM-5) criteria B. Demonstrates impaired daily functioning and maladaptive responses to stress C. Communicates significant distress that has not interfered with important activities D. Employs various defense mechanisms to protect the ego from anxiety

B. Demonstrates impaired daily functioning and maladaptive responses to stress

A client is admitted to a psychiatric unit with the diagnosis of catatonic schizophrenia. Which of the client's neurotransmitters should the nurse expect to be elevated? A. Serotonin B. Dopamine C. Gamma-aminobutyric acid (GABA) D. Histamine

B. Dopamine

Which of the following believed mental illness was curable? A. Benjamin Rush B. Dorothea Dix C. Florence Nightingale D. Linda Richards

B. Dorothea Dix

When working with clients of any culture, which action should the nurse avoid? A. Maintaining eye contact to avoid being rude and making the client uncomfortable B. Assuming that all individuals within a culture or ethnic group are similar C. Supporting the client to participate in cultural and spiritual rituals D. Using an interpreter to obtain health information during an assessment

B. Each client should be treated as an individual and encouraged to participate in his or her cultural and spiritual rituals.

The nurse in the intensive care unit (ICU) is giving report to the nurse on the cardiac step-down unit. The nurse states, "The patient is a 48-year-old woman admitted 3 days ago for chest pain and a stent placement. Vital signs are stable, but I am worried about her stress level. She said she just moved here due to a job transfer and her husband stayed behind to sell the house. She told me they have a high insurance deductible and she is worried about the hospital bill." Which factor has the most significant influence on the client's health? A. Coping skills B. Existing conditions C. Individual vulnerability D. Perceived threat

B. Existing conditions

Which of the following would provide you with the LEAST amount of information about the emotional state of a 20-year-old male who will not respond to any of your questions? A. Respirations B. Hair length C. Facial expression D. Pulse rate

B. Hair length

Which of the following conditions would be LEAST likely to cause an alteration in behavior? A. Brain injury B. Hypertension C. Hypoxemia D. Hypoglycemia

B. Hypertension

The nurse identifies which symptom as typical of the fight-or-flight response? A. Decreased heart rate B. Increased peristalsis C. Increased salivation D. Pupil constriction

B. Increased peristalsis

The nursing instructor described cerebral structures and the "emotional brain" to students. Teaching is effective if the students identify which brain structure? A. Cerebellum B. Limbic system C. Cortex D. Left temporal lobe

B. Limbic system

A 32-year-old woman is speaking to the office nurse at an initial visit. The nurse asked, "What brings you in today?" The woman replied, "I have been having headaches three to four times a week for the past month or so. I'm not sleeping well and feel tired most of the time. I work 60 hours per week and am going through a divorce." The nurse determines the client's symptoms represent which of the following? A. Adaptive coping B. Maladaptive coping C. Problem-solving D. Self-awareness

B. Maladaptive coping

Which mental illness should a nurse identify as being associated with a decrease in prolactin hormone level? A. Major depressive disorder B. Schizophrenia C. Anorexia nervosa D. Alzheimer's disease

B. Schizophrenia

A nurse is educating a patient about the difference between mental health and mental illness. Which statement indicates teaching was effective? A. Mental health is the absence of any stressors. B. Mental health is successful adaptation to stressors in the internal and external environment. C. Mental health is incongruence between thoughts, feelings, and behavior D. Mental health is a diagnostic category in the DSM-5.

B. Mental health is successful adaptation to stressors in the internal and external environment.

The psychiatric nurse is performing the admission assessment of a female client who is being admitted for depression and anxiety. The client reports a long history of excessive alcohol use and the recent loss of her job. When the nurse asks whether she has a religious preference or affiliation, the client states, "I used to believe in God, but I don't anymore. I don't understand how God can allow bad things to keep happening to me." Which of the following nursing diagnoses will the nurse include in the client's care plan? A. Risk for lack of faith B. Risk for spiritual distress C. Risk for impaired religiosity D. Risk for impaired spirituality

B. NANDA describes "risk for spiritual distress" as "vulnerable to an impaired ability to experience and integrate meaning and purpose in life through connectedness within self, literature, nature and/or a power greater than oneself which may compromise health." Risk factors may include substance abuse, depression, anxiety and life changes.

An instructor is teaching nursing students about neurotransmitters. Which term best explains the process by which neurotransmitters released into the synaptic cleft return to the presynaptic neuron? A. Regeneration B. Reuptake C. Recycling D. Retransmission

B. Neurotransmitters are released into the synaptic cleft and returned to the presynaptic neuron by reuptake. Reuptake is the process by which neurotransmitters are stored for reuse.

The school nurse is assessing a female high school student who is distraught because her parents can't afford horseback riding lessons. The nurse recognizes the student's perception is that the problem is: A. Endangering her well-being. B. Personally relevant. C. Based on immaturity. D. Exceeds her capacity to cope.

B. Personally relevant.

Which of the following is the MOST reliable sign of a potentially violent patient? A. History of violence B. Physical activity C. The patient's posture D. Vocal activity

B. Physical activity

A student confides in the school nurse that he is "stressed out" about deciding whether to go to college or work to provide income the family needs. Which coping strategy is the nurse's best recommendation? A. Meditation B. Problem-solving training C. Relaxation D. Journaling

B. Problem-solving training

A teenage boy is attracted to a female teacher. Without objective evidence, a school nurse overhears the boy state, "I know she wants me." This statement reflects which defense mechanism? A. Displacement B. Projection C. Rationalization D. Sublimation

B. Projection

Six months after her husband and children were killed in a car accident, a client is diagnosed with ulcerative colitis. The nurse should recognize that this situation validates which study perspective? A. Neuroendocrinology B. Psychoneuroimmunology C. Diagnostic technology D. Neurophysiology

B. Psychoneuroimmunology

The nurse is interviewing a client with a history of excessive drinking and multiple arrests for impaired driving. The client states, "I work hard to provide for my family. I don't see why I can't drink to relax." The nurse recognizes the client is using which defense mechanism? A. Projection B. Rationalization C. Regression D. Sublimation

B. Rationalization

Which of the following reflects that the nurse understands the concepts of religion and spirituality? A. Religion provides the answers to questions related to the meaning in life. B. An individual may express his or her spirituality through religion. C. Nonattendance to religious services indicates a lack of faith. D. Spirituality is a set of beliefs grounded in the teachings of a spiritual leader.

B. Religion is one way an individual's spirituality may be expressed. Religion, not spirituality, is expressed as a set of beliefs grounded in the teachings of a spiritual leader.

A client was recently admitted to the inpatient unit after a suicide attempt. He has been placed on a tricyclic antidepressant. Which action should the nurse implement to maintain the client's safety when he is discharged? A. Provide a 6-month supply to ensure long-term compliance. B. Provide a 1-week supply of medication with refills authorized only after he visits his provider. C. Encourage him to increase fluid intake to counteract the common side effect of diarrhea. D. Educate him not to eat foods that contain tyramine

B. Suicide risk often increases as antidepressant medication takes effect and the client's level of depression and mood improve. The client may then have increased energy with which to implement a suicide plan. Small number of TCA pills will reduce risk of OD

A patient presents in the emergency department immediately following a shooting incident in a school where she has been teaching. There is no evidence of physical injury, but she appears very hyperactive and talkative. Which are common initial biological responses to stress? Select all that apply. A. Constricted pupils B. Watery eyes C. Unusual food cravings D. Increased heart rate E. Increased respirations

B. Watery eyes D. Increased heart rate E. Increased respirations

A nursing instructor is teaching about suicide in the elderly population. Which information should the instructor include? A. Elderly people use less lethal means to commit suicide. B. While the elderly make up less than 13% of the population, they account for 16% of all suicides. C. Suicide is the second leading cause of death in the elderly. D. It is normal for elderly individuals to express a desire to die because they have come to terms with their mortality.

B. While the elderly make up less than 13% of the population, they account for 16% of all suicides.

A 31-year-old man has signs and symptoms of excited delirium. When assessing his orientation, memory, and ability to concentrate, you should: A. ask him if he knows the date and time and when he last visited his physician. B. ask him simple questions, such as "When did you first notice these feelings?" C. allow him up to 30 seconds to answer a question before asking another one. D. inquire about his medical history, including any medications that he is taking.

B. ask him simple questions, such as "When did you first notice these feelings?"

You arrive at the residence of a young female who is experiencing an apparent psychotic episode. Law enforcement is on scene and has ensured that it is safe. You find the patient sitting in a chair, rocking back and forth. You should: A. remain calm as you gently place your hand on her shoulder. B. clearly identify yourself and your partner and tell her you are there to help. C. reassure her that everything will be okay and ask her if she can walk. D. kneel down in front of the patient and ask her what is troubling her.

B. clearly identify yourself and your partner and tell her you are there to help

You are dispatched to an apartment complex for a young male experiencing an "emotional problem." Law enforcement is at the scene when you arrive. You should: A. request a paramedic unit to chemically sedate the patient. B. confer with a police officer before making patient contact. C. have the police restrain the patient before you assess him. D. enter the scene and ask a police officer about the nature of the problem.

B. confer with a police officer before making patient contact

When restraining a violent patient, it is important to: A. utilize at least 3 people to effectively restrain the patient. B. ensure that you have taken standard precautions first. C. use significant force to effectively subdue the patient. D. avoid talking to the patient during the restraining process.

B. ensure that you have taken standard precautions first

When caring for a patient with a behavioral emergency, it is important for you to: A. spend minimal time at the scene and transport the patient quickly. B. let the patient tell you what happened in his or her own words. C. clearly identify yourself as being in charge and set ground rules. D. qualify the patient's hallucinations in order to gain his or her trust.

B. let the patient tell you what happened in his or her own words

When performing your assessment of a 40-year-old male who is agitated and is displaying bizarre behavior, you should: A. put your hand on the patient's arm or shoulder. B. limit the number of personnel around the patient. C. ask a police officer to assess the patient first. D. have at least four personnel around the patient.

B. limit the number of personnel around the patient

Alzheimer disease is an example of a/an: A. psychological behavioral disorder. B. organic psychiatric disorder. C. functional behavioral disorder. D. psychosocial psychiatric disorder.

B. organic psychiatric disorder

A 78-year old patient presents with delirium. What is your first thought? a) constipation b) hyperthyroidism c) urinary tract infection d) Parkinson's disease

C

Territoriality and density are elements of which of Giger's cultural phenomena? a) Communication b) Social organization c) Space d) Time

C Along with distance, the space various cultures used to communicate, territoriality, the innate tendency to own space and density, the number of people within a given environmental space are concepts within the phenomenon of space.

The client states, "I get into trouble because I respond violently without thinking. That usually gets me into a mess." Which nursing reply is most therapeutic? a) "Everybody loses their temper. It's good that you know that about yourself." b) "I'll bet you have some interesting stories to share about overreacting." c) "Let's explore methods to help you stop and think before taking action." d) "It's good that you are showing readiness for behavioral change."

C Rationale: Helping the client identify appropriate problem-solving behaviors and alternative ways to release tension is a therapeutic nursing intervention. Prevention is the key issue in managing aggressive or violent behavior.

A nursing instructor is teaching about grief resolution. Which student statement indicates a need for further instruction? a) "The grief response can last from weeks to years." b) "Individuals must progress through the grieving process at their own pace." c) "If loss is sudden and unexpected, mourning may take a shorter period of time" d) "Relationship ambivalence and guilt can be linked in the grieving process."

C Rationale: If lost a sudden unexpected, mourning may take a longer, not shorter period of time. Recall the concepts related to grief resolution and the importance of time as it relates to the grieving process.

A pediatric nurse is caring for a 4-year-old child with autism who keeps banging his head against the wall. Which is the appropriate nursing intervention for the child? A) Placing him in restraints until the aggression subsides B) Sedating him with neuroleptic agents C) Having him wear a helmet D) Distracting him with a variety of games and puzzles

C) Having him wear a helmet

When interviewing a client, which nonverbal behavior should a nurse employ? A. Maintaining indirect eye contact with the client B. Providing space by leaning back away from the client C. Sitting squarely, facing the client D. Maintaining open posture with arms and legs crossed

C.

Which client statement indicates the nurse's teaching about the effect of circadian rhythms is effective? A. "When I dream about my mother's horrible train accident, I become hysterical." B. "I get really irritable during my menstrual cycle." C. "I'm a morning person, so I get my best work done in the a.m." D. "Every February, I tend to experience periods of sadness."

C. "I'm a morning person, so I get my best work done in the a.m."

During an intake assessment, a nurse asks both physiological and psychosocial questions. The client angrily responds, "I'm here for my heart, not my head problems." Which is the nurse's best response? A. "It's just a routine part of our assessment. All clients are asked these same questions." B. "Why are you concerned about these types of questions?" C. "Psychological factors, like excessive stress, have been found to affect medical conditions." D. "We can skip these questions, if you like. It isn't imperative that we complete this section."

C. "Psychological factors, like excessive stress, have been found to affect medical conditions."

Devastated by a divorce from an abusive husband, a wife completes grief counseling. Which statement by the wife should indicate to a nurse that the client is in the acceptance stage of grief? A. "If only we could have tried again, things might have worked out." B. "I am so mad that the children and I had to put up with him as long as we did." C. "Yes, it was a difficult relationship, but I think I have learned from the experience." D. "I still don't have any appetite and continue to lose weight."

C. "Yes, it was a difficult relationship, but I think I have learned from the experience."

A minimum of _____ personnel should be used to restrain a 17-year-old, 120-pound, violent patient. A. 1 B. 2 C. 4 D. 3

C. 4

The mental health nurse is conducting an intake interview with a couple seeking marital counseling. The nurse recognizes the husband is using the ego defense mechanism of projection when he exhibits which of the following? A. Stamps his feet and demands his wife honor her vows B. Ignores his wife's continues absence from the home C. Accuses his wife of infidelity and betrayal D. Takes out his frustration by verbally abusing his coworkers

C. Accuses his wife of infidelity and betrayal

A supervisor openly disagrees with an employee's suggestions during a staff meeting. The employee's behavior represents the defense mechanism of displacement when he does which of the following actions? A. Assertively confronts the supervisor B. Abruptly leaves the staff meeting C. Angrily criticizes a coworker D. Takes the supervisor to lunch

C. Angrily criticizes a coworker

The nurse understands that abnormal levels of growth hormone may play a role in which disorder? A. Acute mania B. Schizophrenia C. Anorexia nervosa D. Alzheimer's disease

C. Anorexia nervosa

An unemployed college graduate confides in the clinic nurse that she is experiencing severe anxiety over not finding a teaching position and that she is having difficulty with independent problem-solving. Which nursing intervention is best? A. Encourage her to use other coping mecha- nisms. B. Complete the problem-solving process for her. C. Assist her with the problem-solving process. D. Encourage her to keep a daily journal of feelings.

C. Assist her with the problem-solving process.

Which of the following conditions would be the LEAST likely cause of mental incapacitation? A. Hypoxia and hypercarbia B. CNS depressant drugs C. Chronic hyperthyroidism D. Acute hypoglycemia

C. Chronic hyperthyroidism

Which of the following are included in Jaboda's indicators of mental health? Select all that apply. A. Acceptance B. Creativity C. Environmental mastery D. Fulfillment E. Integration

C. Environmental mastery E. Integration

A nursing student comes down with a sinus infection toward the end of every semester. Which stage of stress is the student most likely experiencing when this occurs? A. Alarm reaction B. Resistance C. Exhaustion D. Fight or flight

C. Exhaustion

Which of the following conditions or situations would result in a psychiatric emergency of an organic etiology? A. Death of a loved one B. Schizophrenia C. Hypoglycemia D. Childhood trauma

C. Hypoglycemia

A lethargic client is diagnosed with major depressive disorder. After taking antidepressant therapy for 6 weeks, the symptoms have not resolved. Which nutritional deficiency should a nurse identify as potentially contributing to the client's symptoms? A. Vitamin A deficiency B. Vitamin C deficiency C. Iron deficiency D. Folic acid deficiency

C. Iron deficiency

7. When a community health nurse arrives at the home of a client diagnosed with bulimia nervosa, the nurse finds the client on the floor unconscious. The client has a history of using laxatives for purging. To what would the nurse attribute this client's symptoms? A. Increased creatinine and blood urea nitrogen (BUN) levels B. Abnormal electroencephalogram (EEG) C. Metabolic acidosis D. Metabolic alkalosis

C. Metabolic acidosis

A client has experienced the death of a close family member and at the same time becomes unemployed. The client's 6-month score on the Recent Life Changes Questionnaire is 110. The nurse: A. Understands the client is at risk for significant stress-related illness. B. Determines the client is not at risk for significant stress-related illness. C. Needs further assessment of the client's coping skills to determine susceptibility to stress-related illness. D. Recognizes the client may view the losses as challenges and perceive them as opportunities.

C. Needs further assessment of the client's coping skills to determine susceptibility to stress-related illness.

A client diagnosed with schizophrenia expresses little emotion and refuses to attend group therapy. The nurse recognizes the client's behavior is influenced by which component of the nervous system? A. Dendrites B. Axons C. Neurotransmitters D. Synapses

C. Neurotransmitters

Which client diagnosis should the nurse associate with a decrease in GABA? A. Alzheimer's disease B. Schizophrenia C. Panic disorder D. Depression

C. Panic disorder

A nurse should recognize that clients who have a history of missed or late medical appointments are most likely to espouse which approach to time and orientation? A. Future‐time oriented B. Past‐time oriented C. Present‐time oriented D. Past‐ and future‐time oriented

C. People from cultures that are present-time oriented may have a history of missed or late medical appointments. They tend to pay less close attention to the clock. This concept of time is casual and focused on the present.

A nurse is performing a mental health assessment on an adult client. According to Maslow's hierarchy of needs, which client action would demonstrate the highest achievement in terms of mental health? A. Maintaining a long-term, faithful, intimate relationship B. Achieving a sense of self-confidence C. Possessing a feeling of self-fulfillment and realizing full potential D. Developing a sense of purpose and the ability to direct activities

C. Possessing a feeling of self-fulfillment and realizing full potential

A fourth-grade boy teases and makes jokes about a cute girl in his class. This behavior should be identified by a nurse as indicative of which defense mechanism? A. Displacement B. Projection C. Reaction formation D. Sublimation

C. Reaction formation

A nurse is assessing a client's response to stress. The nurse concludes that the client is performing a secondary appraisal of a stressful event when the client determines: A. The event is benign. B. The event is irrelevant. C. Resources are available. D. The event is pleasurable.

C. Resources are available.

Which part of the nervous system should the nurse identify as playing a major role during stressful situations? A. Peripheral nervous system B. Somatic nervous system C. Sympathetic nervous system D. Parasympathetic nervous system

C. Sympathetic nervous system

A 30-year-old client seeking therapy states, "My mom cries when she is not included in all my social activities and thinks of my friends as her own." How would the nurse describe the boundaries between this family's parent and child subsystems? A. The boundaries are rigid. B. The boundaries are restructured. C. The boundaries are enmeshed. D. The boundaries are disengaged.

C. The boundaries are enmeshed.

A client diagnosed with bulimia nervosa has been attending a mental health clinic for several months. Which factor should a nurse identify as an appropriate indicator of a positive client behavioral change? A. The client gains 2 pounds in 1 week. B. The client focuses conversations on nutritious food. C. The client demonstrates healthy coping mechanisms that decrease anxiety. D. The client verbalizes an understanding of the etiology of the disorder.

C. The client demonstrates healthy coping mechanisms that decrease anxiety.

Family dynamics are thought to be a major influence in the development of anorexia nervosa. Which information related to a client's home environment should a nurse associate with the development of this disorder? A. The home environment maintains loose personal boundaries. B. The home environment places an overemphasis on food. C. The home environment is overprotective and demands perfection. D. The home environment condones corporal punishment.

C. The home environment is overprotective and demands perfection.

During an intake assessment, the nurse asks a client physiological and psychosocial questions. The client angrily responds, "I'm here for my heart problems, not for my head." Which is the nurse's best response? A. "We ask all clients these questions." B. "Why are you concerned about these questions?" C. "Psychological stress can affect medical conditions." D. "We can skip these questions if you prefer."

C. The nurse should not skip either physiological or psychosocial questions, as this would lead to an inaccurate assessment. The nurse should always attempt to educate the client on the negative effects of excessive stress on medical conditions.

The psychiatric nurse is developing outcomes related to the nursing diagnosis spiritual distress. Which outcome will the nurse designate as a priority for the client? A. Accept and incorporate change into life in a healthy manner. B. Discuss beliefs and values about spiritual and religious issues. C. Identify and understand the meaning and purpose in life. D. Verbalize acceptance of self as a worthwhile human being.

C. This is correct. This outcome addresses spiritual distressed, which is characterized by "an impaired ability to experience and integrate meaning and purpose in life through connectedness within self, literature, nature and/or a power greater than oneself which may compromise health."

A behavioral crisis occurs when: A. an individual reacts violently when faced with danger. B. a person experiences stress without other people's knowledge. C. a reaction to an event interferes with daily living activities. D. an individual suppresses feelings of anger, guilt, or depression.

C. a reaction to an event interferes with daily living activities

What symptom differentiates dissociative fugue from dissociative amnesia?

Clients diagnosed with dissociative fugue are unaware of their memory loss, whereas clients diagnosed with dissociative amnesia are aware of their forgetfulness.

You are dispatched to a residence for an elderly female with "mental status changes." During your assessment of the patient, you should first: A. ask the husband if she has Alzheimer disease. B. perform serial Cincinnati Stroke Scale assessments. C. determine the patient's baseline mental status. D. rule out hypoglycemia by administering glucose.

C. determine the patient's baseline mental status

Behavior is MOST accurately defined as: A. how a person reacts during a violent situation. B. the internal feelings and emotions of a person. C. how a person responds to his or her environment. D. a feeling of internal psychological stability.

C. how a person responds to his or her environment

"Normal" behavior is basically classified as what: A. the patient perceives. B. is agreed upon by psychiatrists. C. is acceptable by society. D. your protocols dictate.

C. is acceptable by society

The nurse is interviewing a newly admitted psychiatric client. Which nursing statement is an example of offering a "general lead"? A. "Do you know why you are here?" B. "Are you feeling depressed or anxious?" C. "Yes, I see. Go on." D. "Can you chronologically order the events that led to your admission?"

C. offering a general lead encourages the client to continue sharing information.

An individual who poses a threat to the safety of his or her family, friends, or the AEMT is experiencing a/an: A. behavioral emergency. B. emotional crisis. C. psychiatric emergency. D. psychological crisis.

C. psychiatric emergency

When interviewing a patient with an emotional crisis, you repeat the patient's words back to him or her, and encourage him or her to expand on these thoughts. This is an example of: A. active listening. B. definitive listening. C. reflective listening. D. appeasing the patient.

C. reflective listening

The single most significant factor that contributes to suicide is: A. marital discord. B. financial setback. C. significant depression.

C. significant depression

The nurse says to a newly admitted client, "Tell me more about what led up to your hospitalization." What is the purpose of this therapeutic communication technique? A. To reframe the client's thoughts about mental health treatment B. To put the client at ease C. To explore a subject, idea, experience, or relationship D. To communicate that the nurse is listening to the conversation

C. the purpose of exploring is to delve further into the subject, idea, experience or relationship. This technique is especially helpful with clients who tend to remain on a superficial level of communication.

The level of force used to restrain a violent patient is dependent upon all of the following factors, EXCEPT the: A. patient's size, gender, strength, and mental status. B. type of abnormal behavior the patient is exhibiting. C. type and dosage of medications he or she is taking. D. force needed to protect the patient from self-injury.

C. type and dosage of medications he or she is taking

Neurological tests have ruled out pathology in a client's sudden lower-extremity paralysis. Which nursing care should be included for this client?

Deal with physical symptoms in a detached manner.

A client with a history of cerebrovascular accident (CVA) is brought to an emergency department experiencing memory problems, confusion, and disorientation. Based on this client's assessment data, which diagnosis would the nurse expect the physician to assign?

Vascular dementia.

Age is the number one risk factor for ischemic strokes. What is the second major risk factor in the older population? a) atrial fibrillation b) methamphetamine c) eclampsia d) hypertension

D

Senescence is the deterioration of the structure and function of body parts as we age. (In other words, your body parts are falling apart). At what age does the process of senescence begin? a) 60 b) 50 c) 40 d) 30

D

Which of the following drugs can contribute to bradycardia? a) digoxin b) metoprolol c) diltiazem d) all of the above e) none of the above

D

The psychiatric nurse is administering medication to a client experiencing a severe depression. The nurse understands that the mechanism of action for some psychoactive medications is related to which process? a) Glutamate inactivation b) Impulse transmission c) Neuropeptide signaling d) Neurotransmitter reuptake

D Rationale: Neurotransmitter reuptake is the process by which neurotransmitters are stored for reuse, a function that holds significance for understanding the mechanism of action of certain psychotropic medications.

A graduate nurse is assigned a client who is mentally handicapped. Which developmental characteristic should the new nurse recognize as typical of a person with severe mental retardation? A) The client can perform some self-care activities independently. B) The client has little, if any, speech development. C) The client's psychomotor skills are usually not affected, except for possible coordination problems. D) The client's wants and needs are often communicated by acting out behaviors.

D) The client's wants and needs are often communicated by acting out behaviors.

A severely depressed, sullen adolescent has been taking fluoxetine (Prozac) for 10 days. During a follow-up visit, the client smile euphorically and states, "I feel so much better now." How might the nurse interpret this behavioral change? a) The Prozac has potentiated serotonin syndrome. b) The medication dosage should be decreased c) The client's behavioral change is normal and expected d) The client may have decided to carry out suicide

D. d) The client has not been taking Prozac long enough to warrant the positive response exhibited. Therefore, the client's euphoria may indicate a decision to carry out a suicide plan. The black box warning label on all antidepressant medications describes this risk and emphasized the need for close monitoring of clients started on these medications. a) Serotonin syndrome occurs when two drugs that potentiate serotonergic neurotransmission are used concurrently causing symptoms that include changes in mental status, restlessness, myoclonus, hyperreflexia, tachycardia, labile, blood pressure, diaphoresis, shivering, and tremors. There is no indication that the client is experiencing serotonin syndrome. b) The client's symptoms do not indicate a change in dosage. c) Mood elevation can be evidenced in 2 to 4 weeks; however, it will take 6 to 8 weeks for major depressive symptoms to subside. The client's behavioral changes would not be normal or expected.

A school nurse is assessing a female high-school student who is overly concerned about her appearance. The client's mother states, "That's not something to be stressed about!" Which response by the nurse is best? A. "Teenagers! They don't know a thing about real stress." B. "Psychological or physical stress occur only when there is a loss." C. "Poor physical condition can interfere with psychological well-being." D. "A threat to self-esteem can result in psy- chological stress."

D. "A threat to self-esteem can result in psy- chological stress."

A potential Olympic figure skater collapses during practice and is hospitalized for severe malnutrition. Anorexia nervosa is diagnosed. Which client statement best reflects the underlying etiology of this disorder? A. "Skaters need to be thin to improve their daily performance." B. "All the skaters on the team are following an approved 1,200-calorie diet." C. "When I lose skating competitions, I also lose my appetite." D. "I am angry at my mother. I can only get her approval when I win competitions."

D. "I am angry at my mother. I can only get her approval when I win competitions."

A client diagnosed with major depressive disorder asks, "What part of my brain controls my emotions?" Which nursing response is best? A. "The occipital lobe judges perceptions as positive or negative." B. "The parietal lobe has been linked to depression." C. "The medulla regulates key biological and psychological activities." D. "The limbic system is largely responsible for one's emotional state."

D. "The limbic system is largely responsible for one's emotional state."

A nursing instructor asks students when diseases of adaptation are likely to occur. Which student response indicates that teaching is effective? A. "When an individual has limited experience dealing with stress." B. "When an individual inherits maladaptive genes." C. "When an individual experiences existing conditions that exacerbate stress." D. "When an individual's physiological and psychological resources are depleted."

D. "When an individual's physiological and psychological resources are depleted."

The emergency department nurse is providing discharge instructions to a 23- year-old man who was injured in a motor vehicle crash. The client stated, "My heart was racing when I saw the car coming through the red light was going to hit me. I didn't know my heart could go that fast!" Which is the nurse's best response? A. "Don't worry, your heart is strong because you are young and in good health." B. "Everyone gets scared when they realize another car is going to hit them." C. "You had a panic attack when you saw the car that hit you was not going to stop." D. "Your body responded to the stress of knowing the car was going to hit you."

D. "Your body responded to the stress of knowing the car was going to hit you."

A physically and emotionally healthy client has just been fired. During a routine office visit, he tells the nurse, "Perhaps this was the best thing to happen. Maybe I'll consider pursuing an art degree." The nurse determines the client perceives the stressor of his job loss as: A. Irrelevant. B. Harm/loss. C. A threat. D. A challenge.

D. A challenge.

Which should the nurse recognize as an example of the defense mechanism of repression? A. A student aware of the need to study for tomorrow's test goes to a movie instead. B. A woman whose son was killed in Iraq does not believe the military report. C. A man who is unhappily married goes to school to become a marriage counselor. D. A woman was raped when she was 12 and no longer remembers the incident.

D. A woman was raped when she was 12 and no longer remembers the incident.

Which of the following is determined by the degree to which thoughts, feelings, and behaviors interfere with an individual's functioning? A. Anxiety B. Defense mechanisms C. Mental health D. Adaptation

D. Adaptation

Which is an example of the ego defense mechanism of regression? A. A mother blames the teacher for her child's failure in school. B. A teenager becomes hysterical after seeing a friend killed in a car accident. C. A woman wants to marry a man exactly like her beloved father. D. An adult throws a temper tantrum when he does not get his own way.

D. An adult throws a temper tantrum when he does not get his own way.

Miller and Rahe (1997) identified a correlation between the effects of life change and illness. This research led to the development of the Recent Life Changes Question- naire (RLCQ). Which principle most limits the effectiveness of this tool? A. Specific physical and psychological illnesses are not identified. B. Numerical values associated with specific life events are randomly assigned. C. Stress is viewed as a solely physiological response. D. An individual's personal perception of the event is excluded.

D. An individual's personal perception of the event is excluded.

A first-time mother is crying and asks the nurse, "How can I go to work if I can't afford child care?" Which is the nurse's initial action to assist the client with problem- solving? A. Determine the risks and benefits for each alternative B. Formulate goals for resolution of the prob- lem C. Evaluate the outcome of the implemented alternative D. Assess the facts of the situation

D. Assess the facts of the situation

The family of a suicidal client is very supportive and requests more facts related to caring for their family member after discharge. Which information should the nurse provide? A. Address only serious suicide threats to avoid the possibility of secondary gain. B. Promote trust by verbalizing a promise to keep suicide attempt information within the family. C. Offer a private environment to provide needed time alone at least once a day. D. Be available to actively listen, support, and accept feelings.

D. Be available to actively listen, support, and accept feelings.

A nurse understands that when a practitioner corrects subluxation by manipulating the vertebrae of the spinal column the practitioner is employing which therapy? A. Allopathic therapy B. Therapeutic touch therapy C. Massage therapy D. Chiropractic therapy

D. Chiropractic therapy

A client is admitted to the emergency department experiencing memory deficits and decreased motor function. What alteration in brain chemistry should the nurse correlate with the presentation of these symptoms? A. Abnormal levels of serotonin B. Decreased levels of dopamine C. Increased levels of norepinephrine D. Decreased levels of acetylcholine

D. Decreased levels of acetylcholine

The nurse recognizes that a decrease in norepinephrine levels plays a significant role in which disorder? A. Mania B. Schizophrenia C. Anxiety D. Major depressive disorder

D. Decreased levels of norepinephrine play a significant role in major depression. The functions of norepinephrine include the regulation of mood, cognition, perception, locomotion, cardiovascular functioning, and sleep and arousal.

The nurse understands that psychotropic medications improve symptoms of mental disorders by acting on which of the following parts of nervous system? A. Basal ganglia B. Hypothalamus C. Interneurons D. Neural synapse

D. Excessive or deficient activity of neurotransmitters influences a variety of cognitive and emotional symptoms. The neural synapse is believed to be the primary site of activity for psychotropic drugs.

What is the purpose of a nurse providing appropriate feedback? A. To give the client good advice B. To advise the client on appropriate behaviors C. To evaluate the client's behavior D. To give the client critical information

D. Feedback should not be used to give advice or evaluate behaviors.

A client who frequently exhibits angry outbursts is diagnosed with Antisocial Personality Disorder. Which appropriate feedback should a nurse provide when this client experiences an angry outburst? A. "Why do you continue to alienate your peers by your angry outbursts?" B. "You accomplish nothing when you lose your temper like that." C. "Showing your anger in that manner is very childish and insensitive." D. "During group, you raised your voice, yelled at a peer, and slammed the door."

D. Giving appropriate feedback involves helping the client consider a modification of behavior. Feedback should give information to the client about how he or she is percieved by others. Feedback should not be evaluative or used to give advice.

A nurse states to a client, "Things will look better tomorrow after a good night's sleep." This is an example of which communication technique? A. The nontherapeutic technique of "giving advice" B. The nontherapeutic technique of "defending" C. The therapeutic technique of "presenting reality" D. The nontherapeutic technique of "giving false reassurance"

D. Giving false reassurance indicates to the client there is no cause for anxiety, thereby devaluing the client's feelings.

During a one-to-one session with a client, the client states, "Nothing will ever get better," and "Nobody can help me." Which nursing diagnosis is most appropriate for a nurse to assign to this client at this time? A. Powerlessness R/T altered mood AEB client statements B. Risk for injury R/T altered mood AEB client statements C. Risk for suicide R/T altered mood AEB client statements D. Hopelessness R/T altered mood AEB client statements

D. Hopelessness R/T altered mood AEB client statements

• Which rationale is given by a nursing instructor to best explain why it is inappropriate to classify Asian Americans as a single culture? A. Extremes of emotional expression prevent accurate assessment of this culture. B. Asian immigrants have been reluctant to participate in large‐ scale cultural research. C. The small size of this subpopulation makes research virtually impossible. D. The Asian American population includes individuals from many different cultures and countries.

D. It is not appropriate to classify Asian Americans as one culture because included are people and descendants from Japan, China, Vietnam, the Phillipines, Thailand, Cambodia, Korea, Laos, etc. There are vast differences in values, religious practices, languages and attitudes within Asian American culture.

The nurse recognizes that a decrease in norepinephrine levels plays a significant role in which disorder? A. Mania B. Schizophrenia C. Anxiety D. Major depressive disorder

D. Major depressive disorder

As part of discharge teaching, which guideline regarding lithium therapy will the nurse plan to include? A. Avoid excessive use of beverages containing caffeine. B. Maintain a consistent sodium intake. C. Consume at least 2500 to 3000 mL of fluid per day. D. All of the above

D. Most Correct. Caffeine, a stimulant, should be limited in clients with mania. Adequate sodium and fluid intake are necessary to prevent lithium toxicity.

A nurse concludes that a restless, agitated client is manifesting a fight-or-flight response. The nurse understands the client's response is associated with which neurotransmitter? A. Acetylcholine B. Dopamine C. Serotonin D. Norepinephrine

D. Norepinephrine

During family counseling a husband states, "Every time my wife and I discuss child discipline, we get into shouting matches." The nurse instructs the couple to shout at each other for 2 weeks on Tuesdays and Thursdays for 30 minutes. What intervention is the nurse using? A. Reframing B. Restructuring the family C. Expressive psychotherapy D. Paradoxical intervention

D. Paradoxical intervention

Which is the most significant consequence of the excessive use of defense mechanisms? A. The superego will be suppressed. B. Emotions will be experienced intensely. C. Learning and the ability to grow will be enhanced. D. Problem-solving will be limited.

D. Problem-solving will be limited.

When under stress, a client routinely uses an excessive amount of alcohol. Finding her drunk, her husband yells at her about the chronic alcohol abuse. Which reaction should the nurse recognize as the use of the defense mechanism of denial? A. Hiding liquor bottles in a closet B. Yelling at their son for slouching in his chair C. Burning dinner on purpose D. Saying to the spouse, "I don't drink too much!"

D. Saying to the spouse, "I don't drink too much!"

Which of the following is LEAST suggestive of a person's intent to commit suicide? A. Giving away personal possessions B. Developing or changing a will C. Procurement of a knife or gun D. Taking time away from a job

D. Taking time away from a job

A client's spouse of 34 years dies unexpectedly. The client cries often and becomes socially isolated. The client's therapist stresses the importance of proper sleep, nutrition, and exercise. What is the best rationale for the therapist's advice? A. An interpersonal approach is indicated for depressed clients. B. Sleep, nutrition, and exercise affect imbalances in neurotransmitters. C. Sleep, nutrition, and exercise will alleviate symptoms of depression. D. The client is susceptible to illness due to effects of stress on the immune system.

D. The client is susceptible to illness due to effects of stress on the immune system.

A nurse is assessing a client who is experiencing occasional feelings of sadness because of the recent death of a beloved pet. The client's appetite, sleep patterns, and daily routine have not changed. How should the nurse interpret the client's behaviors? A. The client's behaviors demonstrate mental illness in the form of depression. B. The client's behaviors are extensive, which indicates the presence of mental illness. C. The client's behaviors are not congruent with cultural norms. D. The client's behaviors demonstrate no functional impairment, indicating no mental illness.

D. The client's behaviors demonstrate no functional impairment, indicating no mental illness.

A client diagnosed with major depressive disorder asks, "What part of my brain controls my emotions?" Which nursing response is best? A. "The occipital lobe judges perceptions as positive or negative." B. "The parietal lobe has been linked to depression." C. "The medulla regulates key biological and psychological activities." D. "The limbic system is largely responsible for one's emotional state."

D. The limbic system is largely responsible for one's emotional state. This system is often called the "emotional brain" and is associated with feelings, sexuality, and social behavior. A. This is incorrect. The occipital lobes are the area of visual reception and interpretation. B. This is incorrect. Somatosensory input (touch, taste, temperature, etc.) occurs in the parietal lobes. C. This is incorrect. The medulla contains vital centers that regulate heart rate and reflexes.

A depressed 21-year-old client has lived with his mother ever since the death of his father 3 years ago. After the client received a college acceptance, the mother repeatedly states, "That's wonderful. I'll be fine all alone." How would the nurse interpret the mother's statements? A. The mother is withholding supportive messages. B. The mother is expressing denigrating remarks. C. The mother is communicating indirectly. D. The mother is using double-bind communication.

D. The mother is using double-bind communication.

Which therapeutic communication technique is being used in this nurse‐ client interaction? Client: "When I get angry, I get into a fistfight with my wife or I take it out on the kids." Nurse: "I notice that you are smiling as you talk about this physical violence." A. Encouraging comparison B. Exploring C. Formulating a plan of action D. Making observations

D. The technique of making observations encourages the client to compare personal perceptions with those of the nurse.

When assessing a patient with bizarre behavior, it is important to ensure your own safety and then: A. talk the patient down to convince him or her to be transported. B. ensure that the patient is in between you and an escape route. C. have law enforcement place the patient in protective custody. D. consider that a medical illness may be an underlying etiology.

D. consider that a medical illness may be an underlying etiology

The two goals to achieve when assessing a patient with a behavioral emergency are to: A. notify law enforcement and transport the patient promptly. B. provide emotional support and diagnose the problem. C. reassure the family and contact the patient's psychiatrist. D. identify life-threats and reduce the stress of the situation.

D. identify life-threats and reduce the stress of the situation

As an AEMT, you may legally restrain a patient if he or she: A. is under the influence of a CNS depressant drug. B. is acutely intoxicated and verbally abusive to you. C. refuses to allow you to begin treatment or transport. D. presents harm to you, your partner, or bystanders.

D. presents harm to you, your partner, or bystanders

General safety guidelines to follow when caring for a patient with a behavioral emergency include all of the following, EXCEPT: A. maintaining a safe distance from the patient. B. being honest and assuring and avoid judging. C. having a predefined definitive plan of action. D. spending as little time as possible at the scene.

D. spending as little time as possible at the scene

Violent or dangerous individuals who do not require medical care should be: A. given a sedative medication. B. transported for psychiatric care. C. subdued with soft restraints. D. taken into police custody.

D. taken into police custody

A violent patient should never be restrained in a prone position because: A. the patient would likely aspirate if he or she vomited. B. suctioning of the airway is not possible in this position. C. the patient will not be able to see what is happening. D. you cannot effectively monitor the patient's airway.

D. you cannot effectively monitor the patient's airway

A client is experiencing progressive changes in memory that have interfered with personal, social, and occupational functioning. The client exhibits poor judgment and has a short attention span. A nurse should recognize these as classic signs of which condition

Dementia

Which of the following is the priority nursing action during the nurse-client relationship?

Establish rapport and develop mutually agreeable treatment goals.

T or F Exercise is a great way to get the bowels moving in the elderly.

FALSE

All of the following are criteria for involuntary commitment for mental health treatment except:

Insufficient funds for food and shelter

A client diagnosed with dissociative identity disorder (DID) switches personalities when confronted with destructive behavior. The nurse recognizes that this dissociation serves which function?

It serves to isolate painful events so that the primary self is protected.

A client diagnosed with Alzheimer's dementia can no longer ambulate, does not recognize family members, and communicates with agitated behaviors and incoherent verbalizations. The nurse recognizes these symptoms as indicative of which stage of the illness?

Late dementia

A client diagnosed with Alzheimer's dementia exhibits progressive memory loss, diminished cognitive functioning, and verbal aggression upon experiencing frustration. Which nursing intervention is most appropriate?

Schedule structured daily routines.

A nurse discovers a client's suicide note that details the time, place, and means to commit suicide. What is the priority nursing intervention and accompanying rationale for this action?

Placing this client on one-to-one suicide precautions, because the more specific the plan, the more likely the client will attempt suicide

Which are examples of primary and secondary gains that clients diagnosed with somatoform pain disorders may experience?

Primary: pain prevents attendance at family reunion; secondary: receives get-well messages

tadalifil (Cialis)

R/T diagnosis of sexual aversion disorder

vardenafil (Levitra)

R/T treatment for Angorganimia and R/T Dx of sexual aversion disorder

A client was diagnosed with depression resulting from the loss of her twin sister in a skiing accident. Her parents reported that all the client has done since the accident was lay in her bed and cry, asking why she survived the accident. The physician prescribed Prozac to treat the depression and suggested that the parents "keep a close eye on her." After a week, the client began to show some signs of improvement, even coming out of her room to eat with the family. After 2 months, the client committed suicide despite seeming to come out of the depression. What is the likeliest reason?

Suicide risk can increase early in treatment with antidepressants.

A client is diagnosed with dissociative identity disorder (DID). What is the primary goal of therapy for this client?

To collaborate among subpersonalities to improve functioning

Which of the following factors can affect the communication process? a) Age b) Culture c) Gender d) Religion e) Sleep patterns

a) Age b) Culture c) Gender d) Religion Rationale: Age influences communication. And example is during adolescence and struggles to establish individual identities generate a unique pattern of communication. Communication has its roots in culture. Culture, mores, norms, ideas, and customs provide the basis for our way of thinking and can affect the communication process. Gender influences communication. Each culture has gender signals that are recognized as either masculine or feminine and affect the communication process. Religion can influence communication. Wearing a clerical collar publicly communicates a minister's mission in life and can affect the communication process. In and of themselves, sleep patterns do not directly affect the communication process.

A malnourish widow of 2 years is angry, obsesses about her loss, and refuses to eat. Which priority nursing diagnosis would be appropriate for this client? a) Complicated grief b) Disturbed sensory perception c) Risk for suicide d) Social isolation

a) Complicated grieving is the priority nursing diagnosis for this client in that after being widowed for 2 years, she remains angry, obsesses with her loss, and refuses to eat. Complicated grieving occurs after the death of a significant other in which the experience of distress accompanying bereavement fails to follow normative expectations and manifest in functional impairment.

A nurse maintains an uncrossed arm and leg posture. This nonverbal behavior is reflective of which letter of the SOLER acronym for active listening? a) S b) O c) L d) E e) R

b) O Rationale: the nurse's uncrossed arm and leg posture is nonverbal behavior that reflects the O in the active-listening acronym SOLER. The acronym SOLER includes sitting squarely facing the client (S), open posture when interacting with the client (O), leaning forward toward the client (L), establishing eye contact (E), and relaxing (R).

opiramate (Topamax)

bipolar, migraines

A suicidal client says to a nurse, "There's nothing to live for anymore." Which is the best nursing reply? a) Why don't you consider doing volunteer work in a homeless shelter?" b) "Let's discuss the negative aspects of your life." c) "Things will look better in the morning." d) "It sounds like you are feeling pretty hopeless."

d) "It sounds like you are feeling pretty hopeless." Rationale: The statement, "It sounds like you are feeling pretty hopeless," helps establish trust and a therapeutic relationship. The nurse is verbalizing the client's implied feelings and allowing the client to validate and explore them.

The family of a suicidal client is supportive and requests more facts related to caring for their family member after discharge. Which information should the nurse provide? a) Address only serious suicide threats to avoid the possibility of secondary gain. b) Promote trust by verbalizing a promise to keep suicide attempt information within the family. c) Offer a private environment to provide needed time alone at least once a day. d) Be available to actively listen, support, and accept the client's feelings.

d) Be available to actively listen, support, and accept the client's feelings. Rationale: Active listening, providing support, and accepting feelings increases the potential that a client would confide suicidal ideations to family members.

Which therapeutic communication technique is being used in this nurse-client interaction? Client: "When I get angry, I get into a fistfight with my wife or I take it out on the kids." Nurse: "I notice that you are smiling as you talk about this physical violence." a) Encouraging comparison b) Exploring c) Formulating a plan of action d) Making observations

d) Making observations Rationale: Encouraging comparison is a therapeutic technique that involves asking the client to compare similarities and differences in ideas, experiences, or interpersonal relationships. Formulating a plan assists the client in making a plan to prevent anger or anxiety from escalating. The nurse is using the therapeutic communication technique of making observations by noting that the client smiles when talking about physical violence. The technique of making observations encourages the client to compare personal perceptions with those of the nurse.

Which symptom should a nurse identify that differentiates clients diagnosed with dementia from clients diagnosed with mood disorders?

impaired memory

The nurse should identify that an example of systematized amnesia is when a client can relate family memories but has

no recollection of a particular brother.

A client diagnosed with dementia is disoriented and ataxic and wanders. Which is the priority nursing diagnosis?

risk for injury

Which symptom exhibited by a client diagnosed with a conversion disorder would predict the poorest prognosis?

seizures

The nurse should assess that a client diagnosed with somatoform pain disorder would experience

severe and prolonged pain in one or more anatomical sites that causes significant distress.

A hungry, homeless client, diagnosed with schizophrenia, refuses to participate in an admission interview. The nurse postpones the admission interview, verbally assures safety, and provides the client with a warm meal. Which of the following does the nurse's action demonstrate?

trust


Related study sets

Block 3 / Unit 3: Domain Name System

View Set

Jackie Robinson Final Paper Terms

View Set

44 & 46: Acute Kidney Injury and Chronic Kidney Disease, Assessment of Urinary System

View Set

Module 2: Reading Assessment Quiz

View Set